Difference between revisions of "2023 AMC 10A Problems/Problem 16"

m (format)
(redirect)
(Tag: New redirect)
 
(8 intermediate revisions by 5 users not shown)
Line 1: Line 1:
==Problem==
+
#redirect[[2023 AMC 12A Problems/Problem 13]]
 
 
In a table tennis tournament every participant played every other participant exactly once. Although there were twice as many right-handed players as left-handed players, the number of games won by left-handed players was <math>40\%</math> more than the number of games won by right-handed players. (There were no ties and no ambidextrous players.) What is the total number of games played?
 
 
 
<math>\textbf{(A) }15\qquad\textbf{(B) }36\qquad\textbf{(C) }45\qquad\textbf{(D) }48\qquad\textbf{(E) }66</math>
 
 
 
==Solution 1 (3 min solve)==
 
We know that the total amount of games must be the sum of games won by left and right handed players. Then, we can write <math>g = l + r</math>, and since <math>l = 1.4r</math>, <math>g = 2.4r</math>. Given that <math>r</math> and <math>g</math> are both integers, <math>g/2.4</math> also must be an integer. From here we can see that <math>g</math> must be divisible by 12, leaving only answers B and D. Now we know the formula for how many games are played in this tournament is <math>n(n-1)/2</math>, the sum of the first <math>n-1</math> triangular numbers. Now setting 36 and 48 equal to the equation will show that two consecutive numbers must equal 72 or 96. Clearly <math>72=8*9</math>, so the answer is \text{\boxed{(B)36}}.
 
 
 
~~ Antifreeze5420
 

Latest revision as of 23:49, 9 November 2023